LSAT and Law School Admissions Forum

Get expert LSAT preparation and law school admissions advice from PowerScore Test Preparation.

General questions relating to LSAT Logical Reasoning.
 jklink94
  • Posts: 6
  • Joined: Sep 04, 2018
|
#57989
Hello!

I have a quick question regarding Prep Test 45 section 2.

The stimulus is as follows: “Art theft from museum is on the rise. Most stolen art is sold to wealthy private collectors. Consequently, since thieves steal what their customers are most interested in buying, museums ought to focus more of their security on their most valuable pieces.”

The correct answer choice is as follows: Art pieces that are not very valuable are not very much in demand by wealthy private collectors.

This sceneraio has occurred to me in a few situations where I would negate both “nots” in the answer choices yielding a phrase that would actually strengthen the argument, and I would mark that as incorrect. As I now know I would only negate the portion “are not very much in demand.” And that would indeed weaken the argument thus being the correct answer choice.

My question is, why is this so? How do I know when to negate only a specific portion of the answer choice if it seems enticing to negate both, of two parts of the answer choice? (such as this situation)

Thank you!
 Brook Miscoski
PowerScore Staff
  • PowerScore Staff
  • Posts: 418
  • Joined: Sep 13, 2018
|
#58013
Hi, JK. I understand that you are wondering how to use the Assumption Negation Technique when there are multiple places you could negate an answer choice. I will take you through this concept in four steps where we explore all the options and their consequences and then discuss what it means about how to use the Assumption Negation Technique.

First, let's look at the ways you could negate Answer Choice B, which is the correct answer. Remember, right now we're not concerned with whether it is correct to negate in this way, just what is possible.

Answer Choice B says "Art pieces that are not very valuable are not very much in demand by wealthy private collectors." Here are the ways we might try to negate it.

1. Art pieces that are very valuable are not very much in demand by wealthy private collectors.
2. Art pieces that are not very valuable are very much in demand by wealthy private collectors.
3. Art pieces that are very valuable are very much in demand by wealthy private collectors.

Second, let's consider how each of those would affect the argument. Before we do, we have to make sure we understand the conclusion, which is that museums can reduce theft (for fencing to wealthy purchasers) by focusing security on the most valuable pieces.

1. Art pieces that are very valuable are not very much in demand by wealthy private collectors.

This weakens the argument. If wealthy purchasers don't want valuable pieces, security should be focused elsewhere.

2. Art pieces that are not very valuable are very much in demand by wealthy private collectors.

This weakens the argument. If wealthy purchasers want non-valuable pieces, focusing on just the valuable pieces won't stop theft.

3. Art pieces that are very valuable are very much in demand by wealthy private collectors.

This strengthens the argument. If wealthy purchasers want valuable pieces, focusing security there makes sense.

Third, let's think about the above and how it resolves your question.

1. Why do the first two options work?

Both of the first two options change the relationship between wealthy purchasers and the kind of art they want to purchase. That allows you to use the Mistaken Negation Technique to see how the argument is affected.

2. Why doesn't the third option work?

The third option doesn't work because it doesn't very much change the relationship between wealthy purchasers and the kind of art that they want to purchase. Think about it this way--as written, Answer Choice B says that wealthy purchasers don't want inexpensive art. Taking out all the "nots" just says that wealthy purchasers want expensive art. That's not much of a change.

Fourth, what is the takeaway from all of this?

When there are two "not" statements, pick one of them and negate. Your goal is to negate the relationship the answer choice proposed, not to take all the "not" statements out. At some point, there may be conditional reasons to pick one part of the answer choice rather than the other, but in general, just remember that to negate the relationship you need to take out one, not both, of the "nots."

This also works when inserting nots--the goal is always to change the relationship in the answer choice, so, pick one part of it to negate.
 jklink94
  • Posts: 6
  • Joined: Sep 04, 2018
|
#58024
Thank you very much! This has greatly helped!

Get the most out of your LSAT Prep Plus subscription.

Analyze and track your performance with our Testing and Analytics Package.